Compleja:Zill-Cap1.3

De luz-wiki
Revisión del 18:09 14 jul 2015 de Ivan de Jesús Pompa García (discusión | contribs.) (Página creada con «Category:Compleja Ejercicios del capítulo 1, sección 3 del libro, A First Course in Complex Analysis with Applications de Zill y Shanahan. ---- == Sección 1.3 == ...»)
(difs.) ← Revisión anterior | Revisión actual (difs.) | Revisión siguiente → (difs.)


Ejercicios del capítulo 1, sección 3 del libro, A First Course in Complex Analysis with Applications de Zill y Shanahan.


Sección 1.3

Ejercicio 1

Escribir el número complejo dado en forma polar utilizando primero una argumento $\theta\neq Arg(z)$ and then using $\theta=Arg(z)$

$2$

Reescribo mi número complejo como $\left(a+bi\right)$, entonces: $z=2=\left(2+0i\right)$

Para expresar un numero complejo en su forma polar es necesario conocer $r=|z|$ y el angulo $\theta$


Para determinar $r$ y $\theta$ solo hay que usar $r=|z|$ y que $tan(\theta)=\frac{b}{a}$


Entonces

$r=|z|=|2+0i|=\sqrt{4}= 2$

y

$tan (\theta) = \frac{0}{2}=0$


$\theta=arctan[0]=0$

$\theta=0$

y luego

$z=r\left[\cos\left(\theta\right)+i\sin\left(\theta\right)\right]$

por lo tanto

$z=2\left[\cos\left(0\right)+i\sin\left(0\right)\right] $


--Emmanuell Castro Flores (discusión) 23:24 15 mayo 2015 (CDT)



Ejercicio 2

   $z=-6$

El número ya esta dado de la forma $a+ib$

Para determinar $r$ y $\theta$ ocupamos $r=|z|$ y $tan(\theta)=\frac{b}{a}$

Por lo que:

$r=|z|=|-6|= 6$

Ahora calculamos el ángulo

$tan(\theta)=\frac{0}{-6}=$

$\theta=arctan[0]=0$

El ángulo $(\theta)$ tiene que estar en el intervalo $-\pi<arg(z)<\pi$ para que sea el argumento de $z$, como el ángulo que se obtuvo esta en este rango,  $\theta$ es el argumento de $z$.

Nancy Martínez Durán (discusión) 16:40 15 mayo 2015 (CDT)



Ejercicio 3

In Problems 1\textendash 10, write the given complex number in polar form first using an argument \textgreek{j} = Arg(z) and then using \textgreek{j} = Arg(z).

traduccion

En los problemas 1-10 , escribir el número complejo dado en forma polar utilizando primero una argumento \textgreek{j} = Arg ( z) y luego utilizando \textgreek{j} = Arg ( z) .

\[ 3.z=3i \]

la forma polar de un numero complejo es una forma de representar a un numero

\[ z=a+bi \]


pero en terminos de su distancia al origen y el angulo que genera respecto al eje real positivo de la siguiente forma

\[ z==|z|\theta \]


para obtener la distancia al origen o modulo del numero se tiene que

\[ |z|=\sqrt{a^{2}+b^{2}} \]


por tanto el modulo del numero dado en el ejercicio sera:

\[ |z|=\sqrt{0^{2}+\left(-3\right)^{2}}=3 \]


para obtener an angulo generado respecto a el eje real positivo se tiene que

\[ \theta=tan^{-1}\left(\frac{b}{a}\right) \]


por tanto el angolo del numero dado en el ejercicio sera:

\[ \theta=tan^{-1}\left(\frac{-3}{0}\right)=\frac{3pi}{2} \]


Entonces sustituyendo los valores el numero complejo en su forma polar es:

\[ z==|3|_{\frac{3pi}{2}} \]


Martin Flores Molina (discusión) 10:29 15 mayo 2015 (CDT)


Hay una indeterminación y además esta igualado a un numero:

$(\frac{-3}{0}\right)=\frac{3pi}{2})$

Nancy Martínez Durán (discusión) 19:55 15 mayo 2015 (CDT)


Ejercicio 4

write the given complex number in polar form first using an argument $\theta\neq Arg(z)$ and then using $\theta=Arg(z)$

$6i$

sabemos que $|z|=r$ , donde r es el radio

$|z|=\sqrt{0^{2}+6^{2}}=\sqrt{36}$

$|z|=6=r$

ahora calculamos el ángulo donde $y=6$ y $x=0$ , pensándolo de una forma gráfica es mas fácil calcular el ángulo con la función Seno, ya que no contamos con una X para calcularlo con un la función Tangente.

$\sin\theta=\frac{y}{r}=\frac{6}{6}=1$

$\arcsin\left(1\right)=\frac{\pi}{2}$

$\theta=\frac{\pi}{2}$

por lo tanto

$z=6\left[\cos\left(\frac{\pi}{2}\right)+i\sin\left(\frac{\pi}{2}\right)\right]$ y $z=6\left[\cos\left(\frac{5\pi}{2}\right)+i\sin\left(\frac{5\pi}{2}\right)\right]$


--Juan Daniel Rivera Bautista (discusión) 17:15 15 mayo 2015 (CDT)


Ejercicio 5

Escribe el numero complejo en su forma polar

$z=1+i$

Para expresar un numero complejo en su forma polar es necesario conocer $r=|z|$ y el angulo $\theta$

Para tener la forma $z=|z|[cos(arg(z)+2{\pi}k)+isin(arg(z)+2{\pi}k)]$

Primero se obtiene los términos que necesitamos como el modulo

$|z|=\sqrt{(z)(z*)}=\sqrt{2}$

Ahora obtendremos el angulo

$\theta=arctan[\frac{y}{x}]=arctan[1]=\frac{\pi}{4}$

Para conocer si $(\theta)$ es el argumento de $z$ o $arg(z)$ debe de satisfacer el intervalo $-\pi<arg(z)<\pi$, como el angulo que obtuvimos se encuentra en el intervalo entonces $\theta$ es el argumento de $z$

Entonces sustituyendo los valores el numero complejo en su forma polar es:

$z=\sqrt{2}[cos(\frac{\pi}{4}+2(\pi)k)+sin(\frac{\pi}{4}+2(\pi)k)]$

El valor de $k$ es arbitrario

Angelina Nohemi Mendoza Tavera (discusión) 10:29 15 mayo 2015 (CDT)


Ejercicio 6

Write the given complex number in polar form first using an argument $\theta\neq Arg\left(z\right)$ and then using $\theta=Arg\left(z\right)$.

6. $5-5i$

Notamos que ya esta de la forma $a+ib$

Para obtener $r$ y $\theta$ sabemos que $r=|z|$ y que $tan(\theta)=\frac{b}{a}$

Entonces

$r=|z|=|5-5i|=\sqrt{50}=5\sqrt{2}$

Y

$tan(\theta)=\frac{-5}{5}=-1$

Entonces $\theta=arctan[-1]=-\frac{\pi}{4}$

Primero

$\theta_{1}=-\frac{\pi}{4}+4\pi=\frac{15\pi}{4}$

Por ello $z_{1}=5\sqrt{2}[cos(\frac{15\pi}{4})+sin(\frac{15\pi}{4})]$

Para $\theta=-\frac{\pi}{4}$

$z=5\sqrt{2}[cos(\frac{-\pi}{4})+sin(\frac{-\pi}{4})]$

--Fernando Vazquez V. (discusión) 02:28 15 mayo 2015 (CDT)


Ejercicio 7

            $z=-\sqrt{3}+i$

El número ya esta dado de la forma $a+ib$

Para determinar $r$ y $\theta$ ocupamos $r=|z|$ y $tan(\theta)=\frac{b}{a}$

Por lo que:

$r=|z|=|-\sqrt{3}+i|=\sqrt{3+1}=2$

Ahora calculamos el ángulo

$tan(\theta)=\frac{1}-\sqrt{3}=$

$\theta=arctan[-\frac{1}\sqrt{3}]=-\frac{\pi}{6}$

El ángulo $(\theta)$ tiene que estar en el intervalo $-\pi<arg(z)<\pi$ para que sea el argumento de $z$, como el ángulo que se obtuvo esta en este rango,  $\theta$ es el argumento de $z$.

Entonces sustituimos los valores anteriores( el angulo y r)

$z={2}[cos(-\frac{\pi}{6}+2(\pi)k)+sin(-\frac{\pi}{6}+2(\pi)k)]$

$k$ es un valor arbitrario

Nancy Martínez Durán (discusión) 16:16 15 mayo 2015 (CDT)


Ejercicio 7

Escribe el numero complejo dado en forma polar usando los argumentos $\theta\neq Arg\left(z\right)$y después usando $\theta=Arg\left(z\right)$

$-\sqrt{3}+i$

Para calcular $r$ y $\theta$ consideramos que $x=-\sqrt{3}$ y $y=1$, donde las operaciones a realizar serian


$r=|z|=\sqrt{x^2+y^2}=\sqrt{(-\sqrt{3})^2+(1)^2}=\sqrt{4}=2$


y para


$\theta=\arctan\left(\frac{y}{x}\right)=\arctan\left(\frac{1}{-\sqrt{3}}\right)=-\frac{\pi}{6}$


Si $x<0$ entonces $\theta=\pi+\arctan\left(\frac{y}{x}\right)=\pi-\frac{\pi}{6}=\frac{5\pi}{6}=\arg\left(z\right)$


La forma polar del numero seria


$z=2\left[\cos\left(\frac{5\pi}{6}\right)+i\sin\left(\frac{5\pi}{6}\right)\right]$


Ahora si consideramos a $\theta=-\pi-\frac{\pi}{6}=-\frac{7\pi}{6}$


$z=2\left[\cos\left(-\frac{7\pi}{6}\right)+i\sin\left(-\frac{7\pi}{6}\right)\right]$

Miguel Medina Armendariz (discusión) 15:32 15 mayo 2015 (CDT)


Ejercicio 8

Escribir el número complejo en su forma polar usando $\Theta=arg\left(z\right)$ , $\theta\neq arg\left(z\right)$

$z=-2-2\sqrt{3}i$

Tenemos que

$x=-2$ , $y=-2\sqrt{3}i$

$r=\left|z\right|=\sqrt{\left(-2\right)^{2}+\left(-2\sqrt{3}\right)^{2}}=4$

Ahora calculamos

$arctan\frac{y}{x}=\theta$

$\theta=arctan\frac{-2\sqrt{3}}{-2}=\frac{\pi}{3}$

ahora tenemos

$\theta_{1}=arg\left(z\right)+2\pi=\frac{\pi}{3}+2\pi=\frac{8\pi}{3}$

Finalmente

$z=4\left[cos\frac{\pi}{3}+isen\frac{\pi}{3}\right]$

$z_{1}=4\left[cos\frac{\pi}{3}+isen\frac{\pi}{3}\right]$

Resuelto por Luis Enrique Martínez Valverde (discusión) 18:07 15 mayo 2015 (CDT)

Nota adicional:

Dado que $-\pi<\dfrac{\pi}{3}<\pi$ se cumple que $\dfrac{\pi}{3}=Arg(z)$. Un ángulo $\theta \neq Arg(z)$ sería:

$arg(z)=Arg(z)+2k\pi$ con $k=1, 2,...$. Si $k=1$:

$z_{2}=4[cos\frac{7\pi}{3}+isen\frac{7\pi}{3}]$

Oscar Javier Gutierrez Varela (discusión) 21:51 15 mayo 2015 (CDT)


Ejercicio 10

En los problemas 1-10, escriba el número complejo dado, en forma polar

primero usando un argumento $\theta\neq Arg\left(z\right)$y después usando $\theta=Arg\left(z\right)$

10.- $\left(\frac{12}{\sqrt{3}+i}\right)$

. Primero reescribimos la expresión dada, utilizando el conjuado de $z$:

$\left(\frac{12}{\sqrt{3}+i}\right)$.$\left(\frac{\sqrt{3}-i}{\sqrt{3}-i}\right)=\left(\frac{12\sqrt{3}-i}{3-i\sqrt{3}+i\sqrt{3}+1}\right)=\left(\frac{12\sqrt{3}-i12}{4}\right)=3\sqrt{3}-i3$

.

Como $x=3\sqrt{3}$, $y=-3$

Podemos obtener:

.

$r=\left|z\right|=\sqrt{\left(3\sqrt{3}\right)^{2}+\left(-3\right)^{2}}=\sqrt{27+9}=6$

Así:

$r=6$

Ahora podemos calcular:

$\frac{y}{x}=\frac{-3}{3\sqrt{3}}=\tan\left(\theta\right)$, $\theta=\arctan\left(-\frac{1}{\sqrt{3}}\right)=-\frac{\pi}{6}$

Pero este valor corresponde a:

$\arg\left(z\right)=-\frac{\pi}{6}$

Para obtener un $\theta_{1}\neq\arg\left(z\right)$calculamos:

$\theta_{1}=\arg\left(z\right)+2\pi=-\frac{\pi}{6}+2\pi=\frac{11}{6}\pi$

Finalmente la expresión buscada es:

$z=6\left[\cos\left(\frac{11}{6}\pi\right)i\sin\left(\frac{11}{6}\pi\right)\right]$

Luego utilizando el argumento principal:

$z=6\left[\cos\left(-\frac{1}{6}\pi\right)i\sin\left(-\frac{1}{6}\pi\right)\right]$

Resuelto por:


--Alejandro Juárez Toribio (discusión) 16:02 14 mayo 2015 (CDT)


Ejercicios 13 y 14

In Problems 13 and 14, write the complex number whose polar coordinates $(r,\theta)$ are given in the form $a + ib$.

13.- $(4,\frac{-5\pi}{3})$

14.- $(2,2)$


Solución:

Dados $(r,\theta)$ se pueden determinar las coordenadas rectangulares como:

\begin{equation} x=rcos\theta \end{equation}

\begin{equation} y=rsin\theta \end{equation}


Dados $(x,y)$ se pueden encontrar el modulo y argumento como:

\begin{equation} r=\sqrt{x^{2}+y^{2}} \end{equation}

\begin{equation} \theta=\acute{a}ngtng\left(\frac{y}{x}\right) \end{equation}


13.

$(4,\frac{-5\pi}{3})$

$x=4cos(\frac{\pi}{3})=2$

$y=4sin(\frac{\pi}{3})=2\sqrt{2}$

Por lo tanto:

$(4,\frac{-5\pi}{3})=(2,2\sqrt{2})$


14.

$(2,2)$

$r=\sqrt{2^{2}+2^{2}}=\sqrt{2(4)}=2\sqrt{2}$

$\theta=\acute{a}ngtng\left(\frac{2}{2}\right)=\frac{\pi}{4}$

Por lo tanto: $(2,2)=(2\sqrt{2},\frac{\pi}{4})$



Ejercicio resuelto por: --Luis Santos (discusión) 15:29 12 mayo 2015 (CDT)


Ejercicios 15 y 16

Escriba el número complejo cuya forma polar esta dada,en la forma $a + ib$ . Usa una calculadora si es necesario.

Problema 15

Para solucionar este problema basta con identificar la forma polar de este numero, esto es $z$ esta de la forma $z=r(\cos{\theta}+i\sin{\theta})$ y de donde fácilmente podemos distinguir r y $\theta$ y pasar ala forma $a + ib$ donde;


Así haciendo los respectivos cálculos tenemos;

así ya podemos expresar el numero complejo de la forma polar en la forma $a+bi$ de la siguiente manera:

Problema 16

este problema es analogo con el problema 15 asi podemos de deducir de manera inmediata que;

asi haciendo los respectivos calculos tendremos;

así ya podemos expresar el numero complejo de la forma polar en la forma $a+bi$ de la siguiente manera:


ejercicios elaborados por --Cristian Alfredo Ruiz Castro (discusión) 19:51 15 mayo 2015 (CDT)


Ejercicio 17 y 18

Encontrar $z$ de la forma $a+bi$ para cada número complejo expresado de la forma polar 17.- $$z=6(\cos({\frac{\pi}{8}})+\sin({\frac{\pi}{8}}))$$ $$a=6\cos({\frac{\pi}{8}})$$ $$b=6\sin({\frac{\pi}{8}})$$ $$a=5.54$$ $$b=2.29$$ $$z=5.54+2.29i$$

18.- $$z=4(\cos({\frac{3\pi}{8}})+\sin({\frac{3\pi}{8}}))$$ $$a=4\cos({\frac{3\pi}{8}})$$ $$b=4\sin({\frac{3\pi}{8}})$$ $$a=2.29$$ $$b=5.54$$ $$z=2.29+5.54i$$ --Manuel Alejandro Chavarría Silva (discusión) 23:28 15 mayo 2015 (CDT)


Ejercicio 19.

Usar $z_{1}z_{2}=r_{1}r_{2}[cos(\theta_{1}+\theta_{2})+isen(\theta_{1}+\theta_{2})]$ y $\frac{z_{1}}{z_{2}}=\frac{r_{1}}{r_{2}}[cos(\theta_{1}-\theta_{2})+isen(\theta_{1}-\theta_{2})]$ para encontrar $z_{1}z_{2}$ y $\frac{z_{1}}{z_{2}}$ . Escribir el número en la forma $a+bi$ .

\[ z_{1}=2[cos(\frac{\pi}{4})+isen(\frac{\pi}{8})] \]


\[ z_{2}=2[cos(\frac{3\pi}{8})+isen(\frac{3\pi}{8})] \]


Entonces:

\[ z_{1}z_{2}=(2)(4)[cos(\frac{\pi}{4}+\frac{3\pi}{8})+isen(\frac{\pi}{8}+\frac{3\pi}{8})] \]


Rsolviendo:

\[ z_{1}z_{2}=8[cos(\frac{5\pi}{8})+isen(\frac{\pi}{2})] \]


Y finalmente, expresando de la forma $a+bi$ tenemos:

$a=8cos(\frac{5\pi}{8})=-.38$

$b=8sen(\frac{\pi}{2})=8$

$z=-.38+8i$

Ahora:

\[ \frac{z_{1}}{z_{2}}=\frac{2}{4}[cos(\frac{\pi}{4}-\frac{3\pi}{8})+isen(\frac{\pi}{8}-\frac{3\pi}{8})] \]


\[ \frac{z_{1}}{z_{2}}=\frac{1}{2}[cos(-\frac{\pi}{8})+isen(-\frac{\pi}{4})] \]


Y finalmente, expresando de la forma $a+bi$ tenemos:

$a=\frac{1}{2}cos(-\frac{\pi}{8})=.4615$

$b=\frac{1}{2}sen(-\frac{\pi}{4})=-\frac{1}{2\sqrt{2}}$

$z=.4615-\frac{1}{2\sqrt{2}}i$

Resuelto por --A. Martín R. Rabelo (discusión) 23:47 12 mayo 2015 (CDT)


Ejercicio 20

 Use (6) and (7) to find $z_{1}z_{2}$ and $\dfrac{z_1}{z_2}$. Write the number in the form $a + ib$ 
 $z_1 = \sqrt{2}[\cos(\dfrac{\pi}{4}) + i\sin(\dfrac{\pi}{4})]$,
 $z_2 = \sqrt{3}[ \cos (\dfrac{\pi}{12}) + i\sin (\dfrac{\pi}{12})]$
 Solución:
 (6):   $ z_{1} z_{2}$ = $r_{1} r_{2}[\cos(\theta_{1} + \theta_{2})+ i\sin(\theta_{1} + \theta_{2})]$
 (7):   $\dfrac{z_1}{z_2}$ = $\dfrac{r_{1}}{r_{2}} [\cos(\theta_{1} - \theta_{2}) + i\sin(\theta_{1} - \theta_{2})]$ 
 Usando (6) para $z_1 z_2$, tenemos, 
 $z_{1} z_{2}$ = $\sqrt{2} \sqrt{3} [\cos(\dfrac{\pi}{4} + \dfrac{\pi}{12}) + i\sin (\dfrac{\pi}{4} + \dfrac{\pi}{12})]$ 
   = $\sqrt{6} [\cos (\dfrac{\pi}{3}) + i\sin (\dfrac{\pi}{3})]$ 
  Como lo queremos en la forma    $ a + ib$, entonces calculamos el valor de a y de b, como sigue, 
  a = $\sqrt{6} \cos (\dfrac{\pi}{3})$ = $\dfrac{\sqrt{6}}{2}$ 
  b = $\sqrt{6} \sin (\dfrac{\pi}{3})$ = $\sqrt{6} \dfrac{\sqrt{3}}{2}$ = $\dfrac{\sqrt{18}}{2}$ = $\sqrt{\dfrac{9}{2}}$ 
  Por lo tanto, 
  $z_{1} z_{2}$ = $\dfrac{\sqrt{6}}{2} + i\sqrt{\dfrac{9}{2}}$.
 Ahora usamos (7) para hallar $\dfrac{z_{1}}{z_{2}}$, entonces, 
 $\dfrac{z_{1}}{z_{2}}$ = $\dfrac{\sqrt{2}}{\sqrt{3}} [\cos (\dfrac{\pi}{4} - \dfrac{\pi}{12}) + i\sin (\dfrac{\pi}{4} - \dfrac{\pi}{12})]$
 = $\sqrt{\dfrac{2}{3}} [\cos (\dfrac{\pi}{6}) + i\sin (\dfrac{\pi}{6})]$ 
 Nuevamente determinamos los valores de a y b. 
 a = $\sqrt{\dfrac{2}{3}} \cos (\dfrac{\pi}{6})$ = $\sqrt{\dfrac{2}{3}} \dfrac{\sqrt{3}}{2}$ = $\dfrac{1}{\sqrt{2}}$ 
 b = $\sqrt{\dfrac{2}{3}} \sin (\dfrac{\pi}{6})$ =$\sqrt{\dfrac{2}{3}} \dfrac{1}{2}$ = $\dfrac{1}{\sqrt{6}}$ 
 Por lo tanto, 
 $\dfrac{z_{1}}{z_{2}}$ = $\dfrac{1}{\sqrt{2}} + \dfrac{1}{\sqrt{6}}i$. 



--Arnold B. Herrera Rubert (discusión) 22:14 15 mayo 2015 (CDT)




En los problemas 25 a 30 emplea la siguiente ecuación para calcular las potencias indicadas.


Ejercicio 25

Calcular Empleando la fórmula propuesta

Calculando primero

Por lo que sustituyendo en la expresión de Moivre´s :

Por lo anterior:

--Pablo (discusión) 09:54 15 mayo 2015 (CDT)



Ejercicio 29

Solución: Empleamos la formula propuesta y vamos simplificando.

Error al representar (error de sintaxis): z^{12}= 2⁶(\cos{\frac{12\pi}{8}}+ i\sin{\frac{12\pi}{8}})
Error al representar (error de sintaxis): z^{12}= 2⁶(\cos{\frac{3\pi}{2}}+ i\sin{\frac{3\pi}{2}})

--Anahi Limas (discusión) 23:08 14 mayo 2015 (CDT)



Ejercicio 30

Calcule la siguiente potencia .

Solución

Usando la fórmula de Moive se tiene

Simplificando términos se obtiene

Elaborado --Ricardo Garcia Hernandez (discusión) 21:21 15 mayo 2015 (CDT)


Ejercicio 31

In Problems 31 and 32, write the given complex number in polar form and in then in the form a + ib.

31. $(\cos(\frac{\pi}{4})+i\sin(\frac{\pi}{4}))^{12}\left[2(\cos(\frac{\pi}{6})+i\sin(\frac{\pi}{6}))\right]$

Considerando la formula de Moivre ; Si $z=r(\cos\theta+i\sin\theta)$ entonces $z^{n}=r^{n}(\cos n\theta+i\sin n\theta)$

Por lo tanto la ecuacion quedaria escrita de la siguiente manera

\[ (\cos\frac{12\pi}{9}+i\sin\frac{12\pi}{9})\left[32(\cos\frac{5\pi}{6}+i\sin\frac{5\pi}{6})\right] \]


Simplificando

\[ 36(\cos\frac{4\pi}{3}+i\sin\frac{4\pi}{3})(\cos\frac{5\pi}{6}+i\sin\frac{5\pi}{6}) \]


Como sabemos, multiplicar complejos es sumar sus angulos y multiplicar sus normas, asi tenemos lo siguiente

\[ 36(\cos\frac{13\pi}{6}+i\sin\frac{13\pi}{6}) \]


considerando a $a=32\cos\frac{13\pi}{6}$ y a $b=32\sin\frac{13\pi}{6}$, tenemos el numero complejo $z=27.71+i16$

Jose Emmanuel Flores Calderón (discusión) 01:00 15 mayo 2015 (CDT)


Ejercicio 32

Resolver para $\dfrac{[8(\cos \frac{3\pi}{8}+i\sin \frac{3\pi}{8})]^3}{[2(\cos \frac{\pi}{16}+i\sin \frac{\pi}{16})]^{10}}$


Decimos que $z_1=8(\cos \frac{3\pi}{8}+i\sin \frac{3\pi}{8})$ y $z_2=2(\cos \frac{\pi}{16}+i\sin \frac{\pi}{16})$.


Se tiene que la fórmula para elevar un número complejo $z$ a su n-ésima potencia es:


$z^n=r^n(\cos n\theta + i\sin n\theta)$


así:


$z_1^3=8^3(\cos 3\frac{3\pi}{8}+i\sin 3\frac{3\pi}{8})=512(\cos \frac{9\pi}{8}+i\sin \frac{9\pi}{8})$


$z_2^{10}=2^{10}(\cos 10\frac{\pi}{16}+i\sin 10\frac{\pi}{16})=1,024(\cos \frac{5\pi}{8}+i\sin \frac{5\pi}{8})$


Dividiendo cantidades:


$\dfrac{z_1^3}{z_2^{10}}=\dfrac{512(\cos \frac{9\pi}{8}+i\sin \frac{9\pi}{8})}{1,024(\cos \frac{5\pi}{8}+i\sin \frac{5\pi}{8})}=\dfrac{(\cos \frac{9\pi}{8}+i\sin \frac{9\pi}{8})}{2(\cos \frac{5\pi}{8}+i\sin \frac{5\pi}{8})}$


Multiplicamos y dividimos por el conjugado del denominador:


$\dfrac{z_1^3}{z_2^{10}}=\dfrac{(\cos \frac{9\pi}{8}+i\sin \frac{9\pi}{8})}{2(\cos \frac{5\pi}{8}+i\sin \frac{5\pi}{8})}\dfrac{2(\cos \frac{5\pi}{8}-i\sin \frac{5\pi}{8})}{2(\cos \frac{5\pi}{8}-i\sin \frac{5\pi}{8})}$


Resolviendo primero la parte superior de la división:


$2(\cos \frac{9\pi}{8}+i\sin \frac{9\pi}{8})(\cos \frac{5\pi}{8}-i\sin \frac{5\pi}{8})=2(\cos \frac{9\pi}{8}\cos \frac{5\pi}{8}-i\cos \frac{9\pi}{8}\sin \frac{5\pi}{8}+i\cos \frac{5\pi}{8}\sin \frac{9\pi}{8}+\sin \frac{9\pi}{8}\sin \frac{5\pi}{8})$


Agrupamos las partes real e imaginaria:


$2[(\cos \frac{9\pi}{8}\cos \frac{5\pi}{8}+\sin \frac{9\pi}{8}\sin \frac{5\pi}{8})+i(\cos \frac{5\pi}{8}\sin \frac{9\pi}{8}-\cos \frac{9\pi}{8}\sin \frac{5\pi}{8})]$


Con las identidades trigonométricas $\sin (a-b)=\sin a \cos b - \sin b \cos a$ y $\cos (a-b)=\cos a \cos b +\sin a \sin b$


$2[\cos (\frac{9\pi}{8}-\frac{5\pi}{8})+i\sin (\frac{9\pi}{8}-\frac{5\pi}{8})]=2(\cos \frac{\pi}{2}+i\sin \frac{\pi}{2})=2i$


Por otro lado, para la parte inferior de la división:

$4(\cos \frac{5\pi}{8}+i\sin \frac{5\pi}{8})(\cos \frac{5\pi}{8}-i\sin \frac{5\pi}{8})=4(\cos^2 \frac{5\pi}{8} + \sin^2 \frac{5\pi}{8})=4$


Sustituyendo ambas cantidades:

                                     $\dfrac{z_1^3}{z_2^{10}}=\dfrac{2i}{4}=\dfrac{1}{2}i$

Oscar Javier Gutierrez Varela (discusión) 19:09 13 mayo 2015 (CDT)



Ejercicio 33

Use la fórmula de Moivre con n=2, encuentre la sidentidades trigonométricas para $\cos\left(2\theta\right)$ y $\sin\left(2\theta\right)$

Definimos:

$z=\cos\left(\theta\right)+i\sin\left(\theta\right)$

Entonces $z^{2}$ puede ser escrito como:

$z^{2}=\cos\left(2\theta\right)+i\sin\left(2\theta\right)=\left(\cos\left(\theta\right)+i\sin\left(\theta\right)\right)^{2}$
$z^{2}=\cos\left(2\theta\right)+i\sin\left(2\theta\right)=\left(\cos\left(\theta\right)\right)^{2}+2i\sin\left(\theta\right)\cos\left(\theta\right)-\left(\sin\left(\theta\right)\right)^{2}$

Igualando parte real con parte real e imaginaria con imaginaria se obtienen:

$\cos\left(2\theta\right)=\left(\cos\left(\theta\right)\right)^{2}-\left(\sin\left(\theta\right)\right)^{2}\;\qquad\;\sin\left(2\theta\right)=2\sin\left(\theta\right)\cos\left(\theta\right)$


--Tlacaelel Cruz (discusión) 21:47 12 mayo 2015 (CDT)


Ejercicio 36

Encontrar un número entero positivo n para el cual la igualdad se cumple en .

Solución

Con y con la fórmula de Moivre

entonces

Por lo que

Proponemos el argumento para el producto de Error al representar (función desconocida «\cdotp»): n\cdotp\theta=2\pi...(1) ; despejando y sustituyendo se tiene que

por lo tanto

Elaborado por --Ricardo Garcia Hernandez (discusión) 22:12 15 mayo 2015 (CDT)


Problema 37

For the complex numbers $z_{1}= -1 and z_{2}= 5 i$ verify that:

\[ (a) Arg (z_{1}z_{2})\neq Arg (z_{1}) + Arg(z_{2})\] \[ (b) Arg(z_{1}/z_{2})\neq Arg(z_{1}) - Arg (z_{2})\]

Dibujamos los números plano complejo para encontrar el argumento \[ Arg(z_{1}) = \dfrac{3}{2} \pi \] \[ Arg (z_{2}) = \pi \]

(a) \[ Arg (z_{1}z_{2})=Arg (\dfrac{3}{2} \pi)(\pi)= 14.80\]

\[ ({3}{2} \pi) + (\pi)= 7.80\] \[ 14.80 \neq 7.80\]


(b) \[ \dfrac{{3}{2} \pi}{\pi} = \dfrac{3}{2}\]

\[ \dfrac{3}{2} \pi - \pi= \dfrac{1}{2}\pi\] \[ \dfrac{3}{2} \neq \dfrac {1}{2}\]

--Esther Sarai (discusión) 00:34 15 mayo 2015 (CDT) Esther Sarai